PT18.S2.Q03 - Citizen of Morresville: Mooresville's current

_kizilbash_kizilbash Member
edited January 2018 in Logical Reasoning 62 karma

Hi everyone I'm having trouble with this question. I think I sort of understand it but if someone can clarify any details Im missing I'd appreciate it.
Basically the citizen states he will do two things to ensure incumbents aren't re-elected. 1) campaign against all these incumbents 2) vote for the incumbent who represents his own neighborhood because she's the only one that knows what she is doing. He then goes on to argue that if everyone in Mooresville follows his example there will be a change in the councils membership.
the question stem then states "assuming that each citizen in Mooresville is allowed to vote only for a city council representative...."
so were proving what must be true for the council membership to change.

A) is true because the citizen in the stimulas is making the exception of voting for an incumbent in his neighborhood that he think will do a good job. But if everyone else from different neighborhoods does the same thing there stands a possibility that all the incumbents will be voted back in and do a bad job all over again in dealing with municipal finances so the voters in answer choice A shouldn't make the same exception that the citizen is making but instead the voters should vote for other representatives, which will substantially change the councils membership.
is this correct and is my reasoning of in any way...
Thanks

https://7sage.com/lsat_explanations/lsat-18-section-2-question-03/

Comments

  • inactiveinactive Alum Member
    12637 karma

    Bumping this to the top!

  • BlindReviewerBlindReviewer Alum Member
    edited January 2018 855 karma

    Yeah you're on the mark -- A is correct because what must be true for the city council to be changed is that not everyone follows the guy's lead and campaigns against everyone else but ultimately votes for the only person they actually have the right to vote for (their neighborhood representative).

    Though I'm kind of new to LR/7Sage, this was a weird question to me because a lot of the evidence needed to answer it is in the question stem itself. Does anyone know if this the case for some questions in other tests as well or did this trend phase out?

  • paul-a-spaul-a-s Member
    7 karma

    One thing that's confusing about this question is that it seems like a combination of a Flaw and an MBT question. On the one hand, the question stem makes it clear that it's MBT. On the other hand, the correct AC implies that the Citizen has made a bad argument. (The Citizen is saying that if everyone follows her lead, there will be change; and AC A says that’s wrong--if everyone follows her lead there will NOT be change. ) I can't think of many if any MBT questions where the answer contradicts something in the stimulus. I guess the giveaway here is that the question stem doesn't actually say "assume the above is true", unlike other MBT questions.

Sign In or Register to comment.